跳到主要内容

基于负相关创新的异方差半参数模型中小波估计量的渐近性质

摘要

考虑异方差半参数回归模型\(y{i}=x{i}\beta+g(t{i})+\varepsilon{i}\),\(i=1,2,\ldot,n),其中β是未知的斜率参数,\(\varepsilon_{i}=\西格玛_{i} e(电子)_{i} \),\(\西格玛^{2}_{i} =f(u_{i})\),\((x_{i},t_{i},u_{i})\)是非随机设计点,\(y{i}\)是响应变量,(f)未知函数定义在闭合区间上吗\([0,1]\),随机错误\(\{e_{i}\}\)均数为零的负相关(NA)随机变量。而的核估计量β,、和(f)在本文中,我们研究了它们的小波估计,并在NA误差假设下导出了这些估计的强相合性。同时,我们还得到了小波估计的Berry–Esséen型界β.

1介绍

考虑以下部分线性或半参数回归模型:

$$y_{i}=x{i}\beta+g(t_{i})+\varepsilon_{i{,\quad1\leqi\leqn$$
(1.1)

哪里β是未知的斜率参数,\(\varepsilon_{i}=\西格玛_{i} e(电子)_{i} \),\(\西格玛^{2}_{i} =f(u_{i})\),\((x_{i},t_{i},u_{i})\)是非随机设计点,\(y{i}\)是响应变量,(f)未知函数定义在闭合区间上吗\([0,1]\)、和\(\{e_{i}\}\)都是随机错误。

众所周知,这个模型(1.1)当错误发生时,许多作者对其特殊情况进行了广泛研究\(e_{i}\)独立同分布(i.i.d.)。例如,当\(\西格玛^{2}_{i} =\西格玛^{2}\),型号(1.1)被简化为通常的部分线性模型,该模型由Engle等人首次引入[1],然后使用各种估计方法来获得中未知量的估计量(1.1)及其渐近性质,(参见[25]). 在混合假设下β衍生于[69]。何时\(\西格玛^{2}_{i} =f(u_{i})\),型号(1.1)成为异方差半参数模型,Back和Liang[10]、张和梁[11]、以及魏和李[12]分别建立了模型的最小二乘估计(LSE)和加权最小二乘估计(WLSE)的强相合性和渐近正态性β,基于的非参数估计(f).如果\(g(t)等于0)\(\西格玛^{2}_{i} =f(u_{i})\),型号(1.1)简化为异方差线性模型;什么时候\(\beta\equiv0\)\(\西格玛^{2}_{i} =f(u_{i})\),型号(1.1)归结为异方差非参数回归模型,其未知量的渐近性质由Robinson研究[13]卡罗尔和哈德尔[14]、梁和齐[15].

近年来,小波技术由于能够适应曲线的局部特征,在统计、工程和技术领域得到了广泛的应用。许多作者考虑使用小波方法估计非参数和半参数模型。参见Antoniadis等人[16]、孙和柴[17],Li等人[1820]、薛[21],Zhou等人[22]等等。

本文考虑模型误差的NA。让我们回顾一下NA随机变量的定义。随机变量的有限集合\({X_{i},1\leqi\leqn\})如果对于所有不相交的子集,则称为负相关(NA)\(A,B\子集\{1,2,\ldot,n\}\),

$$\operatorname{Cov}\bigl(f(X_{i},i\in A),g(X__{j},j\in B)\bigr)\leq0$$

哪里(f)是真正的坐标非递减函数,因此它们的协方差存在。随机变量的无限序列\({X_{n},n\geq1\})如果每个\(第2页),\(X_{1},X_{2},\ldots,X_}n})NA随机变量的定义是由Alam和Saxena介绍的[23]由Joag-Dev和Proschan仔细研究[24]。尽管i.i.d.随机变量是NA,但根据定义,NA变量可以是非i.i.d。由于NA在系统可靠性和多元统计分析中的广泛应用,最近NA的概念受到了广泛关注。我们指的是马图拉[25]和邵[26],等等。

在本文中,我们的目的是推导最小二乘估计,加权最小二乘估计β,及其小波估计的强相合性(f)同时,其小波估计的Berry–Esséen型界β研究了NA随机误差下的异方差半参数模型。

其余部分的结构如下。一些基本假设和估计在第节中列出。 2一些符号和主要结果在第节中给出。 第节提供了主要结果的证明。 4。在附录,给出了一些初步引理。

在整篇论文中,\(C,C_{1},C_}2},\ldots\)表示一些不依赖于n个,在不同的地方可能会有所不同。\(\lfloor x \rfloor)我们表示不超过的最大整数x个;\(((j{1},j{2},\ldots,j{n}))代表的任何排列\((1,2,\ldot,n));\(a{n}=O(b_{n})\)方法\(|a{n}|\leq C|b_{n}|\)、和\(a{n}=o(b{n})\)意味着\(a{n}/b{n}\右箭头0).签署人\(I(A)\)我们表示集合的指示函数A类,\(\varPhi(x)\)是标准正态分布函数,以及\(a^{+}=\最大值(0,a)\),\(a ^{-}=\分钟(0,-a)\).所有限值均作为样本量n个趋向∞,除非另有规定。

2估计值和假设

在模型中(1.1),如果β已知为真参数,则因为\(Ee_{i}=0\),我们有\(g(t_{i})=E(y_{i} -x个_{i} β)\),\(1)因此,自然小波估计

$${g}_{n} (t,β)=\sum_{i=1}^{n}(y_{i} -x个_{i} \β)\int_{A_{i}}E_{m}(t,s)\,ds$$

其中,小波核\(E_{m}(t,s)\)可以由定义\(E_{m}(t,s)=2^{m}\sum_{k\in Z}\phi(2^{m} t-k公司)\φ(2^{m} s-k(平方公里))\),直径是缩放函数,\(m=m(n)>0)是一个整数,仅取决于n个、和\(A{i}=[s_{i-1},s_{i})\)是划分的间隔\([0,1]\)具有\(A_{i}中的t_{i}\),\(i=1,2,\ldot,n)、和\(0\leq t_{1}\leq t_{2}\leq\cdots\leq t_}n}=1\).估算β,我们最小化

$$S(\beta)=\sum_{i=1}^{n}\bigl[y_{i} -x个_{i} \beta-g_{n}(t,\beta)\bigr]^{2}=\sum_{i=1}^{n}(\tilde{y}(y)_{我}-\波浪线{x}_{i} \beta)^{2}$$
(2.1)

最小值为(2.1)被发现为

$$\hat{\beta}_{L}=\sum_{i=1}^{n}\波浪线{x}_{i} \波浪线{y}(y)_{i} /S_{n}^{2}$$
(2.2)

哪里\(\波浪号{x}_{i} =x_{我}-\总和{j=1}^{n} x_{j} \ int_{A_{j}}E_{m}(t_{i},s)\,ds\),\(\波浪号{y}_{i} =y_{我}-\总和{j=1}^{n} 年_{j} \ int_{A_{j}}E_{m}(t_{i},s)\,ds\)、和\(S_{n}^{2}=\sum_{i=1}^{n}\波浪线{x}^{2}_{i} \).估计器\({\beta}_{L}\)被称为伦敦政治经济学院β.

当误差为异方差时,我们根据(f).如果\(\西格玛^{2}_{i} =f(u_{i})\)是已知的,那么\({\beta}_{L}\)修改为WLSE

$$\hat{\beta}_{W}=\sum_{i=1}^{n}a{i}\波浪线{x}_{i} \波浪线{y}(y)_{i} /T_{n}^{2}$$
(2.3)

哪里\(a{i}=\西格玛{i}^{-2}=1/f(u{i})\)\(T_{n}^{2}=\sum_{i=1}^{n} 一个_{i} \波浪线{x}^{2}_{i} \)事实上,(f)未知,必须进行估计。何时\(Ee_{i}^{2}=1\),我们有\(E(y)_{i} -x个_{i} \β-g(t_{i}))^{2}=f(u{i})\)因此(f)可以由定义

$$\帽子{f}_{n} (u{i})=\sum{j=1}^{n}(波浪线{y}(y)_{j}-\波浪线{x}_{j} \hat{\beta}_{L})^{2}\int_{B_{j}}E_{m}(u_{i},s)\,ds$$
(2.4)

哪里\(B_{i}=〔s’_{i-1},s’_{i})\)是划分的间隔\([0,1]\)具有\(B_{i}\中的u_{i{\),\(i=1,2,\ldot,n)、和\(0\lequ{1}\lequ{2}\leq \cdots\lequ}=1\).

为了方便起见,我们假设\(\min_{1\leqi\leqn}|\hat{f}_{n} (u{i})|>0\)因此β

$$\波浪线{\beta}_{W}=\sum_{i=1}^{n}a_{ni}\波浪线{x}_{i} \波浪线{y}(y)_{i} /W_{n}^{2}$$
(2.5)

哪里\(a{ni}=1/\hat{f}_{n} (u{i})\),\(1)、和\(W_{n}^{2}=\sum_{i=1}^{n} 一个_{ni}\波浪线{x}^{2}_{i} \).

我们定义了非参数分量的插件估计量对应于\({\beta}_{L}\),\(测试版)、和\(波浪号{\beta}_{W}\)分别由

$$\帽子{g}_{五十} (t)=sum{i=1}^{n}(y_{i} -x个_{i} {\beta}_{L})\int_{A{i}}E_{m}(t,s)\,ds,\quad\quad\hat{g}_{W} (t)=sum{i=1}^{n}(y_{i} -x个_{i} \hat{\beta}_{W})\int_{A{i}}E_{m}(t,s)\,ds$$

$$\波浪号{g}_{W} (t)=sum{i=1}^{n}(y_{i} -x个_{i} \ tilde{\beta}_{W})\int_{A{i}}E_{m}(t,s)\,ds$$

现在,我们列出一些基本假设,这些假设将在Section中使用。 .

(A0):

\({e_{i},i\geq1\}\)是负相关随机变量序列\(Ee_{i}=0\)\(\sup_{i\geq1}E|E_{i}|^{2+\delta}<\infty\)对一些人来说\(增量>0).

(A1):

存在一个函数小时\([0,1]\)这样的话\(x{i}=h(t{i})+v{i}\)、和

  1. (i)

    \(\lim_{n\rightarrow\infty}n^{-1}\sum_{i=1}^{n} v(v)_{i} ^{2}=\varSigma_{0}\)(\(0<\varSigma_{0}<\infty\));

  2. (ii)

    \(最大{1\leqi\leqn}|v{i}|=O(1)\);

  3. (iii)

    \(limsup_{n\rightarrow\infty}(\sqrt{n}\logn)^{-1}\max_{1\leqm\leqn}|\sum_{i=1}^{m} v(v)_{j_{i}|<\infty\).

(A2):
  1. (i)

    (f),、和小时满足1阶Lipschitz条件\([0,1]\),\(h^{v}中的h\),\(v>3/2),其中\(H ^{v}\)是Sobolev有序空间v(v);

  2. (ii)

    \(0<m_{0}\leq\min_{1\leqi\leqn}f(u_{i})\leq\ max_{1\\leqi\ leqn{f.

(A3):

缩放函数直径第页-常规(第页是一个正整数),满足1阶Lipschitz条件,并且具有紧支撑。此外,\(|\hat{\phi}(\xi)-1|=O(\xi)\)作为\(\xi\rightarrow\infty\),其中ϕ̂是的傅里叶变换直径.

(A4):
  1. (i)

    \(\最大_{1\leq i \ leq n}| s_{i} -秒_{i-1}|=O(n^{-1})\);\(最大值{1\leqi\leqn}|s'_{i} -秒'_{i-1}|=O(n^{-1})\);

  2. (ii)

    存在\(d_{1}>0\)\(d_{2}>0\)这样的话\(\min_{1\leqi\leqn}|t_{i} -吨_{i-1}|\geq d_{1}/n\)\(\min_{1\leqi\leqn}|u_{i} -u个_{i-1}|\geqd_{2}/n\).

(A5):

光谱密度函数\(磅/平方英寸(Ω))属于\(\{e_{i}\}\)满足\(0<C_{1}\leq\psi(\omega)\leqC_{2}<\infty\)对于\(\omega\in(-\pi,\pi]\).

(A6):

存在正整数\(p:=p(n)\),\(q:=q(n)\)、和\(k:=k{n}=\floor\frac{n}{p+q}\floor\)这样,对于\(p+q\leq n\),\(qp^{-1}\leq C<\infty\).

备注2.1

假设(A1)、(A2)和(A5)是近期文献(如Härdle)中常用的标准正则性条件[5],Liang等人[6]、梁和范[7]、张和梁[11]。假设(A3)是小波估计的一般条件。根据伯恩斯坦的分块思想,假设(A6)是一个技术条件,如果第页,q个被合理地选择来证明定理3.3(参见Liang等人[15,27]和Li等人[18,20]).

备注2.2

从(A1)(i)、(iii)、(A2)、(A3)和(A4)可以推断出

$$\开始{聚集}n^{-1}\sum_{i=1}^{n}\波浪线{x}_{i} ^{2}\rightarrow\varSigma_{0};\qquad S_{n}^{-2}\sum_{i=1}^{n}\vert\波浪{x}_{i} \vert\leq C;\\C_{3}\leq n^{-1}\sum_{i=1}^{n}\sigma_{i}^{-2}\波浪线{x}_{i} ^{2}\leq C_{4};\qquad T_{n}^{-2}\sum_{i=1}^{n}\bigl\vert\sigma_{i}^{-2-}\波浪线{x}_{i} \bigr\vert\leq C.\end{已收集}$$
(2.6)

备注2.3

\(tilde{\kappa}(t_{i})=\kappa(t_{i})-\sum_{j=1}^{n}\kappa(t_{j})\int_{A{j}}E_{m}(t_{i},s)\,ds\),套\(\kappa=f,g,\text{或}h\)在假设(A2)(i)、(A3)和(A4)下,通过定理3.2英寸[16]由此可见

$$\sup_{t}\bigl\vert\tilde{\kappa}(t)\bigr\vert=O\bigl(n^{-1}+2^{-m}\biger)$$
(2.7)

备注2.4

根据(A1)(ii)(2.7)、和引理答6在中附录很容易获得

$$\max_{1\leq-i\leq-n}\vert\tilde{x}_{i} \vert\leq\max_{1\leq-i\leq-n}\vert\tilde{h}_{i} \vert+max_{1\leqi\leqn}\vertv_{i}\vert+\max_{1\leqj\leqn}\vert-v_{j}\vert\max_{2\leqi \leqn-}\sum_{j=1}^{n}\biggl\vert\int_{A{j}}E_{m}(t_{i},s)\,ds\biggr\vert=O(1)$$
(2.8)

符号和主要结果

为了说明我们的主要结果,我们引入了以下符号。设置

$$\开始{聚集}\sigma^{2}_{1n}=\operatorname{Var}\Biggl(\sum_{i=1}^{n}\波浪线{x}_{i} \西格玛_{i} e(电子)_{i} \Biggr),\qquad\sigma^{2}_{2n}=\operatorname{Var}\Biggl(\sum_{i=1}^{n}\波浪线{x}_{i} \西格玛^{-1}_{i} e(电子)_{i} \Biggr)、\\varGamma^{2}_{n} (t)=\运算符名称{Var}\Biggl(\sum_{i=1}^{n}\sigma_{i} e(电子)_{i} \nint_{A_{i}}E_{m}(t,s)\,ds\Biggr);\qquad u(q)=\sup_{j\geq1}\sum_{j:\vert j-i\vert\geq}\bigl\vert\operatorname{Cov}r)^{2},\qquad\lambda_{4n}=\frac{2^{m}}{n}\log^{2} n个 ; \\ \lambda{5n}=bigl(2^{-m}+n^{-1}\biger)\log n+\sqrt{n}\bigl^{-1}2^{m} ,\qquad\gamma_{2n}=pn^{-1}2^{m} ,\qquad\gamma{3n}=2^{-m/2}+\sqrt{2^{m}/n}\log n;\\\mu{1n}=\sum{i=1}^{3}\lambda{in}^{1/3}+2\lambada{4n}^{1/3}+\lambda{5n};\qquad\mu{2n}=\sum{i=1}^{2}\gamma{in}^{1/3}+\gamma_{3n}^{(2+\δ)/(3+\δ)};\\\upsilon_{1n}=\lambda_{2n}^{\delta/2}+u^{1/3}(q);\qquad\upsilon_{2n}=\gamma_{2n}^{\delta/2}+u^{1/3}(q)。\结束{聚集}$$

定理3.1

假设(A0)、(A1)(i)、,(A2)(A5)持有.如果 \(2^{m}/n=O(n^{-1/2})\),然后

$${(\mathrm{i})}\quad\hat{\beta}_{L}\rightarrow\beta\quad\\textit{a.s.};\qquad{(\mathrm{ii})}\quad\hat{beta}_{W}\rightarrow\beta\quad\\textit{a.s.}$$
(3.1)

此外,如果 \(max_{1\leqj\leqn}|sum_{i=1}^{n} x个_{i} \nint_{A_{i}}E_{m}(t_{j},s)\,ds|=O(1)\),然后

$${(\mathrm{i})}\quad\max_{1\leqi\leqn}\bigl\vert\hat{g}_{五十} (t{i})-g(t{i})\bigr\vert\rightarrow0\quad\textit{a.s.};\qquad{(\mathrm{ii})}\quad\max{1\leqi\leqn}\bigl\vert\hat{g}_{W} (t{i})-g(t{i})\bigr\vert\rightarrow 0\quad\textit{a.s.}$$
(3.2)

定理3.2

假设(A0)、(A1)(i)、,(A2)(A5)都很满意.如果 \(Ee^{2}_{i} =1\),\(\sup_{i} E类|e_{i}|^{p}<\infty\)对一些人来说 \(p>4\) \(2^{m}/n=O(n^{-1/2})\),然后

$$\begin{collected}{(\mathrm{i})}\quad\vert\hat{\beta}_{L}-\β\vert=o\bigl(n^{-1/4}\bigr);\qquad{(\mathrm{ii})}\quad\max{1\leqi\leqn}\bigl\vert\hat{f}_{n} (u{i})-f(u{i})\bigr\vert\rightarrow0\quad\textit{a.s.};\\{(\mathrm{ii})}\quad\tilde{\beta}_{W}\rightarrow\beta\quad\\textit{a.s.}\end{collected}$$
(3.3)

此外,如果 \(max_{1\leqj\leqn}|sum_{i=1}^{n} x个_{i} \nint_{A_{i}}E_{m}(t_{j},s)\,ds|=O(1)\),然后

$$\max_{1\leqi\leqn}\bigl\vert\tilde{g}_{W} (t_{i})-g(t_{i})\bigr\vert\rightarrow0\quad\textit{a.s.}$$
(3.4)

备注3.1

当随机错误\(\{e_{i}\}\)Chen等人[]已证明(3.1)和(3.3)在类似条件下。由于独立随机样本是NA随机样本的特殊情况,定理3.13.2推广了Chen等人的结果[]。Back和Liang[10]也获得了(3.1)–(3.4)NA样本下未知量的加权估计。

定理3.3

假设(A0)(A6)都很满意.如果 \(\mu_{1n}\rightarrow 0\) \(\upsilon_{1n}\rightarrow0\),那么我们有

$$开始{聚集}{(\mathrm{i})}\quad\sup_{y}\biggl\vert P\biggl(\frac{S_{n}^{2}(\hat{\beta}_{左}-\β)}{\sigma_{1n}}\leqy\biggr)-\varPhi(y)\biggr\vert=O(\mu_{1nC}+\upsilon_{1n});\\{(\mathrm{ii})}\quad\sup_{y}\biggl\vert P\biggl(\frac{T_{n}^{2}(\hat{beta}_{W}-\β)}{\sigma_{2n}}\leqy\biggr)-\varPhi(y)\biggr\vert=O(\mu_{1n}+\upsilon_{1n})。\结束{聚集}$$
(3.5)

推论3.1

假设定理的假设3.3都满足了.如果 \(2^{m}/n=O(n^{-\theta})\),\(u(n)=O(n^{-(1-\theta)/(2\theta-1)}),\(压裂{1}{2}<theta\leq\frac{7}{10}\),然后

$$\beart{collected}{(\mathrm{i})}\fquad\sup_{y}\biggl\vert P\biggl(\frac{S_{n}^{2})(hat{\beta)_{左}-\β)}{\sigma_{1n}}\leqy\biggr)-\varPhi(y)\biggr\vert=O\bigl(n^{\frac{\theta-1}{3}}\bigr);\\{(\mathrm{ii})}\ quad\sup_{y}\ biggl \ vert P\ biggl(\ frac{T_{n}^{2})(\ hat{\beta)_{西}-\β)}{\sigma_{2n}}\leqy\biggr)-\varPhi(y)\biggr\vert=O\bigl(n^{\frac{\theta-1}{3}}\bigr)。\结束{聚集}$$
(3.6)

定理3.4

在定理的假设下3.3,如果 \(\mu_{2n}\rightarrow 0\) \(\upsilon_{2n}\rightarrow0\),那么对于每个 \(t\在[0,1]\中),我们有

$$\sup_{y}\biggl\vert P\biggl$$
(3.7)

哪里 \({g}(t)={g}_{五十} (t)\) \(\帽子{g}_{W} (t)\).

推论3.2

在定理假设下3.4,如果

$$2^{m}/n=O\bigl(n^{-\theta}\bigr),\qquad u(n)=O\bigl(n ^{-(\theta-\rho)/(2\rho-1)}\biger),\quad\frac{1}{2}<\theta<1$$

然后

$$\sup_{y}\biggl\vert P\biggl(\frac{\hat{g}(t)-E\hat}g}$$
(3.8)

4主要定理的证明

定理的证明3.1

我们只是证明(3.1)(ii)作为(3.1)(i) 是类似的。

步骤1。设置\(\波浪号{g}_{i} =g(t_{i})-\sum_{j=1}^{n} 克(t_{j})int_{A_{j}}E_{m}(t_{i},s)\,ds\),\(tilde{varepsilon}_{i}=\varepsilen_{我}-\sum_{j=1}^{n}\varepsilon_{j}\int_{A{j}}E_{m}(t_{i},s)\,ds\)很容易理解

$$\开始{aligned}[b]\hat{beta}_{西}-\beta&=T_{n}^{-2}\Biggl[\sum_{i=1}^{n} 一个_{i} \波浪线{x}_{i} \varepsilon_{我}-\sum_{i=1}^{n} 一个_{i} \波浪线{x}_{i} \Biggl(\sum_{j=1}^{n}\varepsilon_{j}\int _{A_{j}}E_{m}(t_{i},s)\,ds\Biggr)+\sum _{i=1}^{n} 一个_{i} \波浪线{x}_{i} \波浪线{g}_{i} \Biggr]\\&:=A_{1n}-A_{2n}+A_{3n}。\结束{对齐}$$
(4.1)

首先,我们证明\(A_{1n}\右箭头0\)a.s.注意\(A_{1n}=\sum_{i=1}^{n}(T_{n}^{-2}一个_{i} \波浪线{x}_{i} \sigma{i})e_{i}=:\sum{i=1}^{n}c_{ni}e(镍)_{i} \)它由(A1)(i)、(A2)(ii)、(2.6)、和\(2^{m}/n=O(n^{-1/2})\)那个

$$开始{聚集}\max_{1\leqi\leqn}\vertc_{ni}\vert\leq\max_}\frac{verta{i}\tilde{x}_{i} \vert}{T_{n}}\cdot\max_{1\leqi\leqn}\frac{\sigma_{i}}{T_}n}=O\bigl(n^{-1/2}\biger),\\sum_{i=1}^{n} c(c)^{2}_{ni}=T_{n}^{-4}\sum_{i=1}^{n} 一个_{i} \波浪线{x}^{2}_{i} \cdot a_{i}\西格玛^{2}_{i} =O\bigl(n^{-1}\bigr)。\结束{聚集}$$

因此,根据引理A.1款,我们有\(A_{1n}\右箭头0\)a.s.显然,

$$A_{2n}=\sum_{j=1}^{n}\Biggl(\sum_{i=1}^}n}T_{n}^{-2}a_{i} \波浪号{x}_{i} \sigma_{j}\int_{A{j}}E_{m}(t_{i},s)\,ds\Biggr)E_{j}=:\sum_{j=1}^{n} d日_{nj}e_{j} ●●●●$$

通过(A2)(ii),引理答6, (2.6)、和\(2^{m}/n=O(n^{-1/2})\)我们可以获得

$$\begin{聚集}\max_{1\leqj\leqn}\vert d_{nj}\vert\leq\Bigl(\max_}1\leq j\leq n}\sigma_{j}\Bigr)\biggl(\max_1\leq i,j\lequen}\int_{A{j}}\Bigl\vert E_{m}(t_{i},s)\Bigr\vert,ds\biggr)\biggl(t_{n}^{-2}\求和{i=1}^{n}\转换A{i}\波浪线{x}_{i} \vert\Biggr)=O\bigl(n^{-1/2}\bigr),\\begin{aligned}\sum_{j=1}^{n} d日^{2}_{nj}&\leq CT_{n}^{-2}\sum_{j=1}^{n}\sum_{i=1}^}\n}\biggl(int_{A_{j}}E_{m}(t_{i},s)^{n} 一个_{i} \波浪线{x}^{2}_{i} \Biggr)\\&\leq C T_{n}^{-2}\sum_{i=1}^{n}\sum_{j=1}^}n}\biggl(int_{A{j}}E_{m}(T_{i},s)\,ds\Biggr)^{2}=O\bigl(2^{m}/n\biger)=O\bigl(n^{-1/2}\bigr)。\结束{对齐}\结束{聚集}$$

因此\(A_{2n}\右箭头0\)a.s.由引理A.1款显然,来自(2.6)和(2.7)我们获得

$$\vert A_{3n}\vert\leq\Bigl(\max_{1\leqi\leqn}\vert_tilde{g}_{i} \vert\Bigr)\cdot\Biggl(T_{n}^{-2}\sum_{i=1}^{n}\verta{i}\波浪线{x}_{i} \vert\Biggr)=O\bigl(2^{-m}+n^{-1}\biger)\rightarrow0$$

第2步。我们证明了(3.2)(i) ,作为(3.2)(ii)类似。我们可以看到

$$\beart{collected}\max_{1\leq i\leq n}\bigl\vert\hat开始{g}_{五十} (t_{i})-g(t_{i})\bigr\vert\\\quad\leq\max_{1\leqi\leqn}\Biggl\{vert\beta-\hat{beta}_{L}\vert\cdot\Biggl \vert\sum_{j=1}^{n} x个_{j} \int_{A_{j}}E_{m}(t_{i},s)\,ds\Biggr\vert+\vert\tilde{g}_{i} \vert+\Biggl\vert\sum_{j=1}^{n}\sigma_{j} e(电子)_{j} \ int_{A_{j}}E_{m}(t_{i},s)\,ds\Biggr\vert\Biggr\}\\quad:=B_{1n}+B_{2n}+B_{3n}。\结束{聚集}$$

与一起(3.1)(i) 在附加假设下,可以得出以下结论\(B_{1n}\右箭头0\)a.s.我们从(A2)(ii)和引理得到答6那个\(B_{3n}\右箭头0\)a.s.通过应用引理A.2款.因此(3.2)(i) 由证明(2.7). □

定理的证明3.2

步骤1。首先,我们证明(3.3)(i) ●●●●。我们有

$$\开始{aligned}[b]\vert\hat{beta}_{左}-\beta\vert&=S_{n}^{-2}\Biggl[\sum_{i=1}^{n}\波浪线{x}_{i} \varepsilon(瓦雷普西隆)_{我}-\sum_{i=1}^{n}\波浪线{x}_{i} \Biggl(sum_{j=1}^{n}\varepsilon_{j}\int_{A{j}}E_{m}(t_{i},s)\,ds\Biggr)+\sum_{i=1}^}{x}_{i} \波浪线{g}_{i} \Biggr]\\&:=C_{1n}-C_{2n}+C_{3n}。\结束{对齐}$$
(4.2)

请注意

$$C_{1n}=\sum_{i=1}^{n}\bigl(S_{n}^{-2}\波浪线{x}_{i} \sima_{i}\bigr)e_{i}:=\sum_{i=1}^{n} c(c)'_{ni}e(镍)_{我}$$

$$C_{2n}=\sum_{j=1}^{n}\Biggl(\sum_{i=1}^}n}S_{n}^{-2}\tilde{x}_{i} \sigma_{j}\int_{A{j}}E_{m}(t_{i},s)\,ds\Biggr)E_{j}:=\sum_{j=1}^{n}d'_{nj}e_{j} ●●●●$$

类似于(3.1)(ii),我们有

$$开始{聚集}\max_{1\leqi\leqn}\bigl\vertc'_{ni}\bigr\vert\leq\biggl{x}_{i} \vert}{S_{n}}\biggr)\cdot\frac{1}{S_}}=O\bigl(n^{-1/2}\bigr),\qquad\sum_{i=1}^{n} c(c)^{\prime2}_{ni}\leq C\sum_{i=1}^{n}\波浪线{x}_{i} ^{2}/S_{n}^{4}=O\bigl(n^{-1}\biger)(t_{i},S)\bigr\vert\,ds\biggr)\biggl(S_{n}^{-2}\sum_{i=1}^{n}\vert\tilde{x}_{i} \vert\Biggr)=O\bigl(2^{m}/n\bigr)=O\bigl(n^{-1/2}\bigr),\\begin{aligned}\sum_{j=1}^{n} d日^{\prime2}_{nj}&\leq CS_{n}^{-2}\sum_{j=1}^{n}\sum_{i=1}^}\n}\biggl(int_{A_{j}}E_{m}(t_{i},s)\,ds\biggr)^{2}_{i} \Biggr)\\&\leq C S_{n}^{-2}\sum_{i=1}^{n}\sum_{j=1}^}n}\biggl(int_{A{j}}E_{m}(t_{i},S)\,ds\Biggr)^{2}=O\bigl(2^{m}/n\biger)=O\bigl(n^{-1/2}\bigr)。\结束{对齐}\结束{聚集}$$

因此,应用引理A.1款和采取\(α=4\),我们得到\(C_{in}=o(n^{-1/4})\)美国。,\(i=1,2).至于\(C_{3n}\),由\(2^{m}/n=O(n^{-1/2})\), (2.6)、和(2.7)我们很容易看到

$$\vert C_{3n}\vert\leq\Bigl(\max_{1\leqi\leqn}\vert_tilde{g}_{i} \vert\Bigr)\cdot\Biggl(S_{n}^{-2}\sum_{i=1}^{n}\vert\tilde{x}_{i} \vert\Biggr)=O\bigl(2^{-m}+n^{-1}\bigr)=O\bigle(n^{-1-4}\biger)$$

第2步。我们证明了(3.3)(ii)。注意到\(\帽子{f}_{n} (u)=\sum_{i=1}^{n}[\波浪线{x}_{i} (\beta-\hat{\beta}_{L})+\波浪线{g}_{i} +\tilde{\varepsilon}_{i}]^{2}\int_{B_{i{}E_{m}(u,s)\,ds\),我们可以看到

$$\开始{aligned}&\max_{1\leq-j\leq-n}\bigl\vert\hat{f}_{n} (u{j})-f(u{j})\bigr\vert\\&\quad\leq\max_{1\leqj\leqn}\Biggl\vert\sum_{i=1}^{n}\varepsilon^{2}_{i} \nint_{B_{i}}E_{m}(u_{j},s)\,ds-f{n}\varepsilon_{j}\int_{A{j}}E_{m}(t_{i},s)\,ds\Biggr)\Biggr\vert\\&\qquad{}+\max_{1\leqj\leqn}\Biggl\vert\sum_{i=1}^{n},ds\Biggl(sum_{j=1}^{n}\varepsilon_{j}\int_{A{j}}E_{m}(t_{i},s),ds\Bigr){g}_{i} int_{B_{i}}E_{m}(u_{j},s),ds\Biggr\vert\&\qquad{}+2\vert\beta-\hat{beta}_{L}\vert\max_{1\leqj\leqn}\Biggl\vert\sum_{i=1}^{n}\ tilde{x}_{i} tilde{varepsilon}{i}\int_{B_{i}E_{m}(u_{j},s),ds\Biggr\vert+(\beta-\hat{beta}_{L})^{2}\max_{1\leqj\leqn}\Biggl\vert\sum_{i=1}^{n}\ tilde{x}_{i} ^{2}\int_{B_{i}}E_{m}(u_{j},s){x}_{i} \波浪线{g}_{i} \int_{B_{i}}E_{m}(u_{j},s)\,ds\Biggr\vert+\max_{1\leqj\leqn}\Biggl\vert\sum_{i=1}^{n}\tilde{g}^{2}_{i} \ int _{B_{i}}E_{m}(u_{j},s)\,ds\Biggr\vert\\&&\ quad:=\sum_{i=1}^{8} 天_{in}。\结束{对齐}$$

至于\(D_{1n}\),我们有

$$\开始{对齐}D_{1n}\leq{}&\max_{1\leqj\leqn}\Biggl\vert\sum_{i=1}^{n} (f)(u{i})\bigl(e_{i{^{2}-1\大)\int_{B_{i}}E_{m}(u_{j},s)\,ds\Biggr\vert\\&+\max_{1\leqj\leqn}\Biggl\vert\sum_{i=1}^{n} (f)(u{i})\int_{B_{i}}E_{m}(u{j},s)\,ds-f(u}j})\ Biggr\vert\=:{}&D{11n}+D{12n}。\结束{对齐}$$

请注意\(Ee_{i}^{2}=1\),所以\(e_{i}^{2}-1=[(e^{+}_{i})^{2} -E类(e^{+}{i})^{2}]+[(e)^{-}_{i} )^{2} -E类(e)^{-}_{i} )^{2}]:=\xi{i{1}}+\xi{2}}\)、和

$$D_{11n}\leq\max_{1\leqj\leqn}\Biggl\vert\sum_{i=1}^{n}\Biggl}\Biggl(f(u{i})\int_{B_{i}}E_{m}(u{j},s)\,ds\biggr)\xi_{i{2}}\biggr\vert$$

\({\xi{i{1}},i\geq1\}\)\({\xi{i{2}},i\geq1\}\)是平均值为零的NA随机变量,\(\sup_{i} 电子|\xi_{i_{j}}|^{p/2}\leq C\sup_{i} E类|e_{i}|^{p}<\infty\),\(j=1,2).通过(A2)(ii)和引理答6我们有

$$\max_{1\leqi,j\leqn}\biggl\vertf(u_{i})\int_{B_{i{}E_{m}(u{j},s)\,ds\biggr\vert=O\bigl(2^{m}/n\biger)=O\bigl(n^{-1/2}\bigr)$$

$$\max_{1\leq_j\leq-n}\sum_{i=1}^{n}\biggl\vert f(u_{i})\int_{B_{i{}E_{m}(u_j},s)\,ds\biggr\vert=O(1)$$

因此\(D_{11n}\右箭头0\)莱玛的a.sA.2款.签署人(2.7)我们有\(|D_{12n}|=O(2^{-m}+n^{-1})\),所以\(D_{1n}\rightarrow0\)美国。

请注意

$$\vert D_{2n}\vert\leq2\Biggl(\max_{1\leq i\leq n}\Biggl\vert\sum_{j=1}^{n}\sigma)_{j} e(电子)_{j} \int_{A_{j}}E_{m}(t_{i},s)\,ds\Biggr\vert\Biggr)\Biggl(\max_{1\leqj\leqn}\Biggl\vert\sum_{i=1}^{n}\sigma_{i} e(电子)_{i} \int_{B_{i}}E_{m}(u_{j},s)\,ds\Biggr\vert\Biggr)$$

类似于\(D_{11n}\),我们获得\(D_{2n}\rightarrow0\)莱玛的a.sA.2款.

应用假设,从(2.6), (2.7), (3.3)(i) ,引理A.2款、和引理答6由此可见

$$开始{aligned}&\vertD_{3n}\vert\leq\max_{1\leqi\leqn}\Biggl(\sum_{j=1}^{n}\sigma_{j} e(电子)_{j} \int_{A_{j}}E_{m}(t_{i},s)\,ds\Biggr)^{2}\max_{1\leqj\leqn}\sum_{i=1}^{n}\biggl\vert\int_{B_{i}}E_}leq{}&2\Bigl(\max_{1\leqi\leqn}\vert\tilde{g}_{i} \vert\Bigr)\cdot\Biggl(\max_{1\leq-j\leq-n}\Biggl\vert\sum_{i=1}^{n}{\varepsilon}_{i}\int_{B_{i{}}E_{m}(u_{j},s)\,ds\Biggr\vert\Biggr)+2\Bigl(\max_1\leq-i\leqn-}\vert\tilde{g}_{i} \vert\Bigr)\\&\cdot\Biggl(\max_{1\leqi\leqn}\Biggl\vert\sum_{k=1}^{n}\varepsilon_{k}\int_{A_{k}}E_{m}{i}}E_{m}(u_{j},s)\,ds\Biggr\vert\Biggr)\\rightarrow{}&0\quad\mbox{A.s.},\end{aligned}\\&\vertD_{6n}\vert\leq(\beta-\hat{\beta}_{L})^{2}\cdot\biggl(\max_{1\leqi,j\leqn}\int_{B_{i}}\bigl\vertE_{m}(u_{j},s)\bigr\vert\,ds\biggr)\biggl(\sum_{i=1}^{n}\tilde{x}_{i} ^{2}\Biggr)=o(1)\quad\mbox{a.s.},\\&\vert D_{7n}\vert\leq2\vert\beta-\hat{beta}_{L}\vert_cdot\Bigl(\max_{1\leqi\leqn}\vert\tilde{g}_{i} \vert\Bigr)\cdot\biggl(\max_{1\leqi,j\leqn}\int_{B_{i}}\bigl\vertE_{m}(u_{j},s)\Bigr\vert\,ds\biggr)\biggl(\sum_{i=1}^{n}\vert\tilde{x}_{i} \vert\Biggr)\rightarrow0\quad\mbox{a.s.},\\&\vert D_{8n}\vert\leq\Bigl(\max_{1\leqi\leqn}\vert_tilde{g}_{i} \vert\Bigr)^{2}\cdot\Biggl(\max_{1\leqj\leqn}\sum_{i=1}^{n}\Biggl\vert\int_{B_{i}}E_{m}(u_{j},s)\,ds\biggr\vert\biggr)\rightarrow0,\\&\vertD_{5n}\vert\leq2\vert\beta-\hat{beta}_{L}\vert\cdot\ Biggl{1\leqj\leqn}\sqrt{\Biggl(\sum_{i=1}^{n}\波浪线{x}^{2}_{i} \Biggr)\sum_{i=1}^{n}\tilde{\varepsilon}^{2}_{i} \biggl(\int_{B_{i}}E_{m}(u_{j},s)\,ds\biggr)^{2}}\biggr)。\结束{对齐}$$

为了证明这一点\(D_{5n}\右箭头0\)美国,这足以表明

$$\max_{1\leqj\leqn}\sum_{i=1}^{n}\tilde{\varepsilon}^{2}_{i} \biggl(\int_{B_{i}}E_{m}(u_{j},s)\,ds\biggr)^{2}=O\bigl(n^{-1/2}\biger)\quad\mbox{a.s.}$$
(4.3)

至于(4.3),我们可以分开

$$\begin{collected}\max_{1\leqj\leqn}\sum_{i=1}^{n}\tilde{\varepsilon}^{2}_{i} \biggl(\int_{B_{i}}E_{m}(u_{j},s)\,ds\biggr)^{2}\\quad\leq\max_{1\leq j\leq n}\sum_{i=1}^{n}{\varepsilon}^{2}_{i} \biggl(int_{B_{i}}E_{m}(u_{j},s)\,ds\biggr biggl(sum_{k=1}^{n}{\varepsilon}_{k}\int_{A{k}}E_{m}(t_{i},s),ds\biggr)\biggl(\int_{B_{i}}E_{m}(u_{j},s)\,ds\biggr)^{2}\biggr)\cdot\biggl(\sum_{k=1}^{n}{\varepsilon}_{k}\int_{A_{k}}E_{m}(t_{i},s)\,ds\biggr)^{2}\\quad:=D_{51n}+D_{52n}+D_{53n}。\结束{聚集}$$

通过引理A.2款答6,自\(2^{m}/n=O(n^{-1/2})\),我们有

$$\begin{aligned}&\begin}aligned{}\vertD_{52n}\vert\leq{}&2\Biggl(\max_{1\leqi\leqn}\Biggl\vert\sum_{k=1}^{n}{\varepsilon}_{k}\int_{A{k}}E_{m}(t_{i},s)\,ds\Biggr\vert\Biggr)\cdot\Biggl(\ max_{1\leqi,j\ leqn}\int_{B_{i}}\bigl\vertE_{m}(u_{j},s)\bigr\vert\,ds\Biggr)\\&\cdot\Biggl(\max_{1\leqj\leqn}\Biggl\vert\sum_{i=1}^{n} {\varepsilon}_{i}\nint_{B_{i{}E_{m}(u_{j},s),ds\Biggr\vert\Biggr)\\={}&o\bigl(n^{-1/2}\biger)\quad\mbox{a.s.},\end{aligned}\\&\begin{alinged}\vertD_{53n}\vert\leq{}&\Biggl(max_{1\leqi\leqn}\Biggl\ vert\sum_{k=1}^{n}{\varepsilon}_{k}\int_{a{k}}E_{m}(t_{i},s)\,ds\Biggr\vert^{2}\Biggr)\cdot\Biggl(\max_{1\leqi,j\leqn}\int_{B_{i}}\bigl\vertE_{m}(u_{j},s)\bigr\vert\,ds\biggr)\&\cdot\Biggl(\max_{1\leqj\leqn}\sum_{i=1}^{n}\Biggl\vert\int_B_{i}}E_{m{(u{j},s)\,ds\biggr\vert\biggr)\\={}&o\bigl(n^{-1/2}\biger)\quad\mbox{a.s.}\end{aligned}\end{aligned}$$

请注意

$$开始{aligned}\vertD_{51n}\vert\leq{}&\max_{1\leqj\leqn}\Biggl\vert\sum_{i=1}^{n}\sigma^{2}_{i} \xi_{i_{1}}\biggl(\int_{B_{i}}E_{m}(u_{j},s)\,ds\biggr)^{2}\biggr\vert+\max_{1\leqj\leqn}\Bigl\vert\sum_{i=1}^{n}\sigma^{2}_{i} \xi_{i_{2}}\biggl(\int_{B_{i}}E_{m}(u_{j},s)\,ds\biggr)^{2}\biggr\vert\\&+\max_{1\leqj\leqn}\Bigl\vert\sum_{i=1}^{n}\sigma^{2}_{i} Ee公司^{2}_{i} \biggl(int_{B_{i}}E_{m}(u_{j},s)\,ds\biggr)$$

哪里\(\xi{i{1}}=[(e^{+}{i})^{2} -E类(e^{+}_{i})^{2}]\)\(xi{i{2}}=[(e^{-}_{i} )^{2} -E类(e)^{-}_{i} )^{2}]\)是具有零均值的NA随机变量。类似于\(D_{11n}\),我们得到\(D'_{51n}=o(n^{-1/2})\)a.s.和\(D''_{51n}=o(n^{-1/2})\)a.s.由引理A.2款另一方面,

$$\bigl\vert D''{51n}\bigr\vert\leq\biggl(\max_{1\leqj\leqn}\sigma^{2}_{i} \int_{B_{i}}\bigl\vertE_{m}(u_{j},s)\bigr\vert\,ds\biggr)\Biggl$$

因此我们证明了\(D_{51n}=O(n^{-1/2})\)a.s.这就完成了(3.3)(ii)。

步骤3。接下来,我们证明(3.3)(iii)通过(A2)(ii)和(3.3)(ii)。何时n个足够大,很容易理解

$$0\leqm'_{0}\leq\min_{1\leqi\leqn}\hat{f}_{n} (u{i})\leq\max{1\leqi\leqn}\hat{f}_{n} (u_{i})\leq M'_{0}\leq\infty$$
(4.4)

\(C_{5}\leqW_{n}^{2}/n\leqC_{6}\)、和\(W^{-2}_{n} \sum_{i=1}^{n}|a_{ni}\波浪线{x}_{i} |\leq C\)。因此,我们有

$$\vert\tilde{\beta}_{无}-\β\vert\leq\frac{n}{W_{n}^{2}}\Biggl\vert\frac{1}{n}\sum_{i=1}^{n}a_{ni}\波浪线{x}_{i} \ tilde{\varepsilon}_{i}\Biggr\vert+\frac{1}{W_{n}^{2}}\Bigl\vert\sum_{i=1}^{n}a_{ni}\ tilde{x}_{i} \波浪线{g}_{i} \Biggr\vert=:\frac{n}{W_{n}^{2}}E_{1n}+E_{2n}$$
(4.5)

与一起(2.7)和(4.5),我们得到

$$\begin{collected}\vertE_{2n}\vert\leq\Bigl(\max_{1\leqi\leqn}\vert_tilde{g}_{i} \vert\Bigr)\cdot\Biggl(W_{n}^{-2}\sum_{i=1}^{n}\verta_{ni}\tilde{x}_{i} \vert\Biggr)\rightarrow0,\\开始{aligned}\vertE_{1n}\vert\leq{}&\Biggl\vert\frac{1}{n}\sum_{i=1}^{n}(a_{ni}-a_{i} )\波浪号{x}_{i} \tilde{\varepsilon}_{i}\Biggr\vert+\Biggl\vert\frac{1}{n}\sum_{i=1}^{n}a_{i}\tilde{x}_{i} \ tilde{\varepsilon}_{i}\Biggr\vert\\\leq{}&\Biggl\vert\frac{1}{n}\sum_{i=1}^{n}(a_{ni}-a_{i} )\波浪号{x}_{i} {\varepsilon}_{i}\Biggr\vert+\Biggl\vert\frac{1}{n}\sum_{i=1}^{n}(a_{ni}-a_{i} )\波浪号{x}_{i} Biggl(sum_{j=1}^{n}\varepsilon_{j}\int_{A{j}}E_{m}(t_{i},s)\,ds\Biggr){x}_{i} \ tilde{\varepsilon}_{i}\Biggr\vert\\=:{}&E_{11n}+E_{12n}+E2{13n}。\结束{对齐}\结束{聚集}$$

我们从引理中知道A.2款那个

$$\开始{对齐}[b]n^{-1}\sum_{i=1}^{n} 电子_{i} ^{2}&=n^{-1}\sum_{i=1}^{n}\bigl[\bigl(e^{+}_{i}\biger)^{2} -E类\bigl(e^{+}_{i}\bigr)^{2}\biger]+n^{-1}\sum_{i=1}^{n}\bigl[\bigl^{-}_{i} \更大)^{2} -E类\比格尔^{-}_{i} \bigr)^{2}\bigr]+n^{-1}\sum_{i=1}^{n} Ee公司^{2}_{i} \\&=O(1)\quad\mbox{a.s.}\end{aligned}$$
(4.6)

应用引理A.2款并结合(4.4)–(4.6)带有(3.3)(ii),我们获得

$$\begin{collected}\vertE_{11n}\vert\leq\biggl(\max_{1\leqi\leqn}\frac{\vert\hat{f}_{n} (u{i})-f(u{i})\vert}{\hat{f}_{n} (u{i})f(u{i})}\biggr)\sqrt{\Biggl(\frac{1}{n}\sum{i=1}^{n}\波浪线{x}^{2}_{i} \Biggr)\Biggl(\frac{1}{n}\sum_{i=1}^{n}{e}^{2}_{i} \Biggr)}\rightarrow0\quad\mbox{a.s.},\\开始{aligned}\vert E_{12n}\vert&\leq\biggl(\max_{1\leqi\leqn}\frac{\vert\hat{f}_{n} (u{i})-f(u{i})\vert}{\hat{f}_{n} (u{i})f(u{i})}\biggr)\cdot\Biggl(\frac{1}{n}\sum_{i=1}^{n}\vert\tilde{x}_{i} \vert\Biggr)\cdot\Biggl(\max_{1\leq i\leq n}\Biggl\vert\sum _{j=1}^{n}\varepsilon\{j}\int _{A_{j}}E_{m}(t_{i},s)\,ds\Biggr\vert\Biggr)\\&&\rightarrow0\quad\mbox{A.s.}\end{aligned}\end{collected}$$

至于\(E_{13n}\),我们有\(E_{13n}=\压裂{T_{n}^{2}}{n}(|A{1n}+A{2n}|)\右箭头0\)因此\(E_{1n}\右箭头0\)美国。 □

证明(3.4)与的相似(3.2)(i) ,因此我们省略了它。

定理的证明3.3

我们只是证明(3.5)(i) ,作为(3.5)(ii)类似。从定义\({\beta}_{L}\)我们有

$$\开始{aligned}[b]S_{n}^{2}(\hat{\beta}_{左}-\β)&=\sum_{i=1}^{n}\波浪线{x}_{i} \西格玛_{i} e(电子)_{我}-\sum_{i=1}^{n}\波浪线{x}_{i} \Biggl(\sum_{j=1}^{n}\西格玛_{j} e(电子)_{j} \nint_{A_{j}}E_{m}(t_{i},s)\,ds\Biggr)+\sum_{i=1}^{n}\波浪线{x}_{i} \波浪线{g}_{i} \\&:=L_{1n}-L_{2n}+L_{3n}。\结束{对齐}$$
(4.7)

设置\(Z_{ni}=frac{tilde{x}_{i} \西格玛_{i} e(电子)_{i} }{\西格玛{1n}}\),我们采用伯恩斯坦的大块和小块程序。\(y{nm}=sum{i=k{m}}^{k{m{+p-1}Z{ni}),\(y'{nm}=sum{i=l{m}}^{l{m{+q-1}Z{ni}),\(y'{nk+1}=sum{i=k(p+q)+1}^{n} Z_{ni}\),\(k{m}=(m-1)(p+q)+1),\(l{m}=(m-1)(p+q)+p+1),\(m=1,2,\ldot,k).然后

$$\西格玛^{-1}_{1n}左_{1n}:=\波浪{左}_{1n}=\sum_{i=1}^{n}Z_{ni}=\sum _{m=1}^{k} 年_{nm}+\总和{m=1}^{k} 年'{nm}+y'{nk+1}=:L_{11n}+L_{12n}+L_{13n}$$

我们观察到

$$\开始{对齐}L_{2n}&=\sum_{i=1}^{n}\Biggl[\tilde{h}_{i} +伏_{我}-\Biggl(\sum_{k=1}^{n} v(v)_{k} \int_{A_{k}}E_{m}(t_{i},s)\,ds\Biggr)\Biggr]\cdot\Biggl(\sum_{j=1}^{n}\sigma_{j} e(电子)_{j} \ int_{A_{j}}E_{m}(t_{i},s)\,ds\Biggr)\\&=\sum_{i=1}^{n}\波浪线{h}_{i} \Biggl(\sum_{j=1}^{n}\sigma_{j} e(电子)_{j} \nint_{A_{j}}E_{m}(t_{i},s)\,ds\Biggr)+\sum_{i=1}^{n} v(v)_{i} \Biggl(\sum_{j=1}^{n}\西格玛_{j} e(电子)_{j} \int_{A_{j}}E_{m}(t_{i},s)\,ds\Biggr)\\&\quad-\sum_{i=1}^{n}\Biggl(\sum_{k=1}^{n} v(v)_{k} \int_{A_{k}}E_{m}(t_{i},s)\,ds\Biggr)\Biggl(\sum_{j=1}^{n}\sigma_{j} e(电子)_{j} \ int_{A_{j}}E_{m}(t_{i},s)\,ds\Biggr)\\&:=L_{21n}+L_{22n}-L_{23n}。\结束{对齐}$$

所以我们可以写

$$\frac{S_{n}^{2}(\hat{\beta}_{左}-\β)}{\西格玛{1n}}=L_{11n}+L_{12n}+L_{13n}+\西格马{1n{^{-1}(L_{21n}+L_{22n}-L_{23n}+L_{3n})$$

通过应用引理A.4款我们有

$$\开始{对齐}[b]&\sup_{y}\biggl\vert P\biggl(\frac{S_{n}^{2})_{左}-\β)}{\sigma_{1n}}\leqy\biggr}^{1/3}\biger)\\&\qquad{}+P\bigl(\sigma_{1n}^{-1}\vertL_{21n}\vert>\lambda_{3n}^{1/3}\bigr)+P\bigl(\sigma_{1n}^{-1}\vert L_{22n}\vert>\lambda_{4n}^{1/3}\bigr)+P\bigl(\sigma _{1n}^{-1}\vert L_{23n}\vert>\lambda_{4n}^{1/3}\bigr)\\&\qquad{}+\frac{1}{\sqrt{2\pi}}\Biggl(\sum_{k=1}^{3}\lambda_{kn}^{1/3}+2\lambda_{4n}^{1/3}\Biggl gr)+\frac{1}{\sqrt{2 \pi}}\sigma\{1n}^{-1}\vert L_{3n}\vert\\&&quad=\sum_{k=1}^{8} 我_{kn}。\结束{对齐}$$
(4.8)

因此,为了证明(3.5)(i) ,这足以表明\(\sum_{k=2}^{8} 我_{kn}=O(\mu_{1n})\(I{1n}=O(\upsilon_{1nneneneep+\lambda^{1/2}_{1n}+\lampda^{1/2}_{2n})这里我们需要以下Abel不等式(参见Härdle et al[5]). \(A_{1},\ldots,A_{n})\(B_{1},\ldots,B_{n})(\(B_{1}\geq B_{2}\geq\cdots\geq B_)是两个实数序列,让\(S_{k}=\sum_{i=1}^{k} A类_{i} \),\(M_{1}=\min_{1\leqk\leqn}S_{k}\)、和\(M_{2}=max_{1\leqk\leqn}S_{k}).然后

十亿美元_{1} M(M)_{1} {k=1}^{n} A类_{k} B类_{k} \列克B_{1} M(M)_{2}. $$
(4.9)

请注意

$$\sigma_{1n}^{2}=E\Biggl(\sum_{i=1}^{n}\波浪线{x}_{i} \西格玛_{i} e(电子)_{i} \Biggr)^{2}=\ int _{-\pi}^{\pi}\psi(\omega)\Biggl\vert\sum _{k=1}^{n}\波浪{x}_{k} \西格玛_{k} e(电子)^{-ik\omega}\Biggr\vert^{2}\,d\omega$$

$$\varGamma_{n}^{2}(t)=E\Biggl(\sum_{i=1}^{n}\sigma)_{i} e(电子)_{i} \ int_{A_{i}}E_{m}(t,s)\,ds\Biggr)^{2}=\int_{-\pi}^{\pi}\psi(\omega)\Biggl\vert\sum_{k=1}^{n}\sigma_{k}\int_A_{k{}}E_ m}$$

根据(A2)(ii)、(A5)、(2.6)、和引理答6由此可见

$$\开始{对齐}和C_{7}n\leq C_{1}\sum_{i=1}^{n}\波浪线{x}_{i} ^{2}\leq\sigma_{1n}^{2{\leqC_{2}\sum_{i=1}^{n}\波浪线{x}_{i} ^{2}\leq C_{8}n,\end{对齐}$$
(4.10)
$$\开始{对齐}和C_{9}\sum_{i=1}^{n}\biggl(\int_{A_{i}}E_{m}(t,s)\,ds\biggr)^{2}\leq\varGamma_{n}^{2{(t)\leqC_10}\sum _{i=1}^{n}\bigl(int_{A_{i}E_{m}(t,s)\,ds\biggr)^{2}=O\bigl(2^{m}/n\bigr)。\结束{对齐}$$
(4.11)

步骤1。我们首先证明\(\sum_{k=2}^{8} 我_{kn}=O(μ_{1n})\).使用引理答3, (2.7)、和(2.8),来自(A0)(i),(A1)–(A6)(4.10)、和(4.11)由此可见

$$\开始{聚集}我_{2n}\leq\fracc{E\vertL_{12n}\vert^{2}}{\lambda_{1n}^{2/3}}\leq \frac{C}{n\lambda_{1n{2/3}}\sum_{m=1}^{k}\sum_{i=L_{m}}^{L_{m}+q-1}\ tilde{x}_{i} ^{2}\西格玛^{2}_{i} \leq\frac{Ckq}{n\lambda{1n}^{2/3}}\leq C\lambda{1n}^{1/3},\\i_{3n}\leq\frac{E\vert L_{13n}\vert ^{2}}}{\lambda{2n}^{2/3}}\leq\frac{C}{n\lambda{2n}^{2/3}}}\sum _{i=k(p+q)+1}^{n}\tilde{x}_{i} ^{2}\西格玛^{2}_{i} \leq\frac{Cp}{n\lambda_{2n}^{2/3}}\leq-C\lambda_{2n{1/3},\\begin{对齐}I_{4n}&\leq\frac{\sigma{1n}^{-2}东\vert L_{21n}\vert^{2}}{\lambda_{3n}^{2/3}}\leq\frac{C}{n\lambda_{3n{2/3}}\sum_{j=1}^{n}\Biggl(\sum_{i=1}^}n}\tilde{h}_{i} \int_{A_{j}}E_{m}(t_{i},s)\,ds\Biggr)^{2}\sigma^{2}_{j} \\&\leq\frac{C}{n\lambda_{3n}^{2/3}}\Bigl(\max_{1\leqi\leqn}\vert\tilde{h}_{i} \vert^{2}\Bigr)\cdot\sum_{j=1}^{n}\Biggl(\max_{1\leqj\leqn}\sum_{i=1}^}\n}\int_{A_{j}}\bigl\vertE_{m}(t_{i},s)\Bigr\vert\,ds\Biggr)^{2{\\leq\frac{C(2^{-m}+n^{-1})^{2\vertE_m}\lambda_{3n}^{2/3}}\leq C\lambda _{3n}^{1/3},\end{aligned}\\begin{对齐}I_{5n}&\leq\frac{\sigma{1n}^{-2}E\vert L_{22n}\vert^{2}}{\lambda_{4n}^{2/3}}\leq\frac{C}{n\lambda{4n{2/3}}\sum_{j=1}^{n}\Biggl(\sum_{i=1}^{n} v(v)_{i} \int_{A_{j}}E_{m}(t_{i},s)\,ds\Biggr)^{2}\sigma^{2}_{j} \\&\leq\frac{C}{n\lambda_{4n}^{2/3}}\max_{1\leqi,j\leqn}\biggl(int_{A{j}}\bigl\vertE_m}(t_{i},s)\bigr\vert\,ds\biggr)\cdot\max_1\leqi\leqn}\sum_{j=1}^{n}\biggl\vert\int_{A{j}}\bigl\vert E_{m}(t_{i},s)\bigr\vert\,ds\biggr\vert\cdot\biggl(\max_{1\leq-m\leq-n}\biggl\vert\sum_{i=1}^{m} v(v)_{j_{i}}\Biggr\vert\Biggr)^{2}\\&\leq\frac{C(2^{m} n个^{-1}\log^{2} n个)}{\lambda_{4n}^{2/3}}\leq C\lambda_{4n}^{1/3},\end{aligned}\\begin{alinged}I_{6n}&\leq\frac{\sigma_{1n}^{-2}东\vert L_{23n}\vert^{2}}{\lambda_{4n}^{2/3}}\leq\frac{C}{n\lambda_{4n{2/3}}\sum_{j=1}^{n}\Biggl(\sum_{L=1}^{n} v(v)_{l} \int_{A{l}}E_{m}(t_{i},s)^{2}_{j} \\&\leq\frac{C}{n\lambda_{4n}^{2/3}}\max_{1\leqi,j\leqn}\biggl(int_{A_{j}}\bigl\vertE_m}(t_{i},s)\bigr\vert\,ds\biggr)\cdot\max_1\leqi\leqn}\sum_{j=1}^{n}\biggl\vert\int_A_{j}}E_{m}(t_{i},s)\,ds\biggr\vert\&\quad\cdot\biggl(\max_{1\leq-l\leq-n}\sum_{i=1}^{n}\biggl\vert\int_{A{l}}E_{m{,ds\biggr\vert\max_{1\leqm\leqn}\Biggl\vert\sum_{i=1}^{m} v(v)_{j_{i}}\Biggr\vert\Biggr)^{2}\\&\leq\frac{C(2^{m} n个^{-1}\log^{2} n个)}{\lambda_{4n}^{2/3}}\leq C\lambda_{4n}^{1/3}。\结束{对齐}\结束{聚集}$$

至于\(I_{8n}\),我们有

$$\开始{对齐}I_{8n}&=\sigma_{1n}^{-1}\Biggl\vert\sum_{i=1}^{n}\波浪线{x}_{i} \波浪线{g}_{i} \Biggr\vert\leq\frac{C}{\sqrt{n}}\Biggl(\Biggl\vert\sum_{i=1}^{n} v(v)_{i} \波浪号{g}_{i} \Biggr\vert+\Biggl\vert\sum_{i=1}^{n}\波浪线{h}_{i} \波浪线{g}_{i} \Biggr\vert+\Biggl\vert\sum _{i=1}^{n}\波浪号{g}_{i} \sum_{j=1}^{n} v(v)_{j} \nint_{A_{j}}E_{m}(t_{i},s)\,ds\Biggr\vert\Biggr)\\&\leq\frac{C}{\sqrt{n}}\max_{1\leqi\leqn}\vert\tilde{g}_{i} \vert\max_{1\leq-m\leq-n}\Biggl\vert\sum_{i=1}^{m} v(v)_{j_{i}}\Biggr\vert+\frac{Cn}{\sqrt{n}}\max_{1\leqi\leqn}\vert\tilde{h}_{i} \vert\max_{1\leq-i\leq-n}\vert\tilde{g}_{i} \vert\\&\quad+\frac{C}{\sqrt{n}}\max_{1\leqi\leqn}\vert\tilde{g}_{i} \vert\cdot\max_{1\leqj\leqn}\sum_{i=1}^{n}\int_{A_{j}}\bigl\vertE_{m}(t_{i},s)\bigr\vert\,ds\cdot\max_{1\\leqm\leqn}\Biggl\vert\sum_{i=1}^{m} v(v)_{j_{i}}\Biggr\vert\\&\leq C\bigl(2^{-m}+n^{-1}\bigr)\bigle(\log n+\sqrt{n}\bigl.(2^}-m}+n ^{-1{\bigr.)\biger)=C\lambda_{5n}。\结束{对齐}$$

因此,根据之前的估计,我们得出\(\sum_{k=2}^{8} 我_{kn}=O(\mu_{1n}).

第2步。我们核实\(I{1n}=O(λ.让\({eta{nm}:m=1,2,\ldots,k\})是具有相同分布的独立随机变量\(y{nm}\),\(m=1,2,\ldot,k).设置\(H_{n}=\sum_{m=1}^{k}\eta_{nm}\)\(s_{n}^{2}=\sum_{m=1}^{k}\operatorname{Var}(y_{nm})\).遵循Liang和Li定理2.1的证明方法[27]和Li等人[20],我们很容易看到

$$\boot{aligned}[b]I_{1n}&=\sup_{y}\bigl\vert P(L_{11n}\leq y)-\varPhi(y)\bigr\vert\\&&\leq\sup_{y}\bigl\vert P(L_{11n}\leq y)-P(H_{n}\leq y)\bigr\vert\\&&quad+\sup_{y}\bigl\vert P(H_{n}\leq y)-\varPhi(y/s_{n})\bigr \vert+\sup_{y}\bigl\vert\varPhi(y/s_{n})-\varPhi(y)\bigr\vert\\&:=I_{11n}+I_{12n}+I_{13n}。\结束{对齐}$$
(4.12)

(i) 我们评估\(s_{n}^{2}\).注意到\(s_{n}^{2}=E{L}^{2}_{11n}-2\sum_{1\leqi<j\leqk}\操作符名{Cov}(y_{ni},y_{nj})\(波浪号{左}_{1n}^{2}=1\),我们可以

$$\bigl\vert E(L_{11n})^{2}-1\bigr\vert\leq C\bigl(\lambda_{1n}^{1/2}+\lambda _{2n}^}_1/2}\bigr)$$
(4.13)

另一方面,从(A1),(A2)(4.10)、和(2.8)由此可见

$$开始{对齐}[b]\biggl\vert\sum_{1\leqi<j\leqk}\operatorname{Cov}(y_{ni},y_{nj}^{k_{j}+p-1}\垂直\波浪线{x}_{s} \波浪线{x}_{t} \sigma_{s}\sigma _{t}\vert\cdot\bigl\vert\operatorname{Cov}(e_{s},e_{t})\bigr\vert\&\leq Ckpn^{-1}u(q) \leq铜(q)。\结束{对齐}$$
(4.14)

因此,从(4.13)和(4.14)由此可见\(|s{n}^{2}-1|\leq C(\lambda_{1n}^{1/2}+\lambda_{2n}^}{1/2{+u(q)).

(ii)应用Berry–Esséen不等式(见Petrov[28],定理5.7),用于\(增量>0),我们得到

$$\sup_{y}\bigl\vert P(H_{n}/s_{n{}\leqy)-\varPhi(y)\bigr\vert\leq C\sum_{m=1}^{k}\bigr(E\vert y_{nm}\vert^{2+\delta}/s_}n}^{2+/delta}\biger)$$
(4.15)

按引理A.3从(A0)、(A1)、(A2)、(4.10)、和(2.8)我们可以推断出

$$\开始{对齐}[b]\sum_{m=1}^{k} 电子\vert y_{nm}\vert ^{2+\delta}&=\sum_{m=1}^{k} E类\Biggl\vert\sum_{j=k_{m}^{k_{m}+p-1}Z_{ni}\Biggr\vert^{2+\delta}\&&\leqC\sigma_{1n}^{-(2+\delta)}\sum_{m=1}^{k}\Biggl\{sum_{i=k_{m}}^{k_{m}+p-1}E\vert\tilde{x}_{i} \西格玛_{i} e(电子)_{i} \vert^{2+\delta}+\Biggl[\sum_{i=k_{m}}^{k_{m}+p-1}E(\波浪线{x}_{i} \西格玛_{i} e(电子)_{i} )^{2}\Biggr]^{1+\delta/2}\bigr\}\\&\leq C\bigl。\结束{对齐}$$
(4.16)

\(s_{n}\右箭头1\)由(4.13)和(4.14),来自(4.15)和(4.16)我们很容易看到\(I_{12n}\leq C\lambda^{delta/2}_{2n}\)。请注意\(I_{13n}=O(|s_{n}^{2}-1|)=O(\lambda_{1n}^{1/2}+\lambda_{2n}^}{1/2{+u(q)).

(iii)接下来,我们评估\(I_{11n}\).让\(\varphi_{1}(t)\)\(瓦尔斐{2}(t))是的特征函数\(L_{11n}\)\(H_{n}\)分别是。因此应用Esséen不等式(见Petrov[28],定理5.3),对于任何\(T>0\),我们有

$$开始{对齐}[b]&\sup_{t}\bigl\vert P(L_{11n}\leqt)-P t}\bigl\vert P(H_{n}\lequ+t)-P。\结束{对齐}$$
(4.17)

从引理答5和(4.14)由此可见

$$\开始{aligned}\bigl\vert\varphi_{1}(t)-\varphi_}(t)\bigr\vert&=\Biggl\vert E\exp\Biggl(\mathrm{i} t吨\sum_{m=1}^{k}y_{nm}\Biggr)-\prod_{m=1{^{k{E\exp{(\mathrm{i} t吨y_{nm})}\Biggr\vert\\&&\leq4t^{2}\sum_{1\leq i<j\leq k}\sum_{s_{1}=k_{i}^{k_{i}+p-1}\sum_{t_{1}=k_{j}^{k_{j}+p-1}\bigl\vert\operatorname{Cov}(Z_{ns_{1}},Z_{nt_{1}})\bigr\vert\\&&\leq4Ct^{2} 单位(q) ,\结束{对齐}$$

这意味着

$$I’_{11n}=\int_{-T}^{T}\biggl\vert\frac{\varphi_{1}(T)-\varphi_{2}(T)}{T}\biggr\vert\,dt\leq-Cu(q)T^{2}$$
(4.18)

因此(4.15)和(4.16)我们有

$$开始{对齐}[b]&\sup_{t}\bigl\vert P(H_{n}\leq t+u)-P u}{s{n}}\biggr)\biggr\vert\\&\qquad+\sup_{t}\bigl\vert P\biggl(\frac{H_{n}{s_{n{}}\leq\frac}{t}{s_n}}\ biggr}\biggr)\biggr\vert+\sup_{t}\biggl\vert\varPhi\biggl(\frac{t+u}{s_{n}}\bigr)-\varPhi\ biggl gr)-\varPhi(t)\biggr\vert+\sup_{t}\biggl\vert\varPhi\biggl(\frac{t+u}{s_{n}}\bigr)-\varPhi\ biggl\biggr\vert\\&\quad\leq C\biggl(\lambda_{2n}^{delta/2}+\biggl\vert\frac{u}{s_{n}}\biggr\vert\biggr)\leq C\ bigl。\结束{对齐}$$
(4.19)

发件人(4.19)由此可见

$$I''{11n}=T\sup_{T}\int_{vert u\vert\leq C/T}\bigl\vert P(H_{n}\leq T+u)-P(H_}n}\leq T)\bigr\vert,du\leq C\bigl(\lambda_{2n}^{delta/2}+1/T\bigr)$$
(4.20)

组合(4.17), (4.18)带有(4.20)和选择\(T=u^{-1/3}(q)\),我们很容易看到\(I_{11n}\leqC(u^{1/3}(q)+\lambda_{2n}^{delta/2}).所以\(I{1n}\leq C(λ.这就完成了定理的证明3.3从步骤1和步骤2开始。 □

推论的证明3.1

在定理中3.3,选择\(p=\floor n^{\theta}\floor),\(q=地板n^{2\theta-1}\rfloor),\(增量=1),何时\(1/2<theta\leq7/10),我们有\(\mu_{1n}=O(n^{-(θ-1)/3})\(upsilon_{1n}=O(n ^{-(θ-1)/3}).因此(3.6)直接遵循定理3.3. □

定理的证明3.4

我们只证明了\({g}(t)={g}_{L} (t)\),作为\({g}(t)={g}_{W} (t)\)是类似的。

根据的定义\(\帽子{g}_{五十} (t)\)我们很容易看到

$$\开始{aligned}\varGamma_{n}^{-1}(t)\bigl(\hat{g}_{五十} (t)-E\hat{g}_{五十} (t)\bigr)={}&\varGamma_{n}^{-1}^{n} x个_{i} (E\hat{\beta}_{左}-\beta)\nint_{A_{i}}E_{m}(t,s)\,ds\Biggr)\\&+\varGamma_{n}^{-1}(t)\Biggl(\sum_{i=1}^{n} x个_{i} (\beta-\hat{\beta}_{L})\int_{A{i}}E_{m}(t,s)\,ds\Biggr)\\:={}&J{1n}+J{2n}+J{3n}。\结束{对齐}$$

设置\(Z'_{ni}=frac{sigma_{i} e(电子)_{i} \int_{A_{i}}E_{m}(t,s)\,ds}{\varGamma_{n}(t)}\)。类似于\(\波浪号{左}_{1n}\),我们可以分开\(J_{1n}\)作为\(J{1n}=sum{i=1}^{n}Z'{ni}:=J{11n}+J{12n}+J{13n}),其中\(J{11n}=sum{m=1}^{k}chi{nm}),\(J{12n}=sum{m=1}^{k}\chi'{nm}),\(J{13n}=chi'{nk+1}),\(chi{nm}=sum{i=k{m}}^{k{m{+p-1}Z'{ni}),\(chi'{nm}=sum{i=l{m}}^{l{m{+q-1}Z'{ni}),\(chi'{nk+1}=sum{i=k(p+q)+1}^{n} Z'_{ni}\),\(k{m}=(m-1)(p+q)+1),\(l{m}=(m-1)(p+q)+p+1),\(m=1,2,\ldot,k).

应用引理A.4款,我们有

$$\开始{对齐}[b]&\sup_{y}\biggl\vert P\biggl(\frac{\hat{g}_{五十} (t)-E\hat{g}_{五十} (t)}{\varGamma_{n}(t){\leqy\biggr)-\varPhi(y)\biggr\vert\\&\quad\leq\sup_{y}\bigl\vert P(J_{11n}\leqy)-\valPhi(y)\bigr\vert+P\bigl(\vert J_{12n}\vert>\gamma_{1n}^{1/3}\biger)+P\bigl{2n}^{1/3}\biger)\\&\qquad+\frac{vertJ{2n{vert}{sqrt{2\pi}}+P\bigl(vertJ}3n}\vert>\gamma{3n}^}(2+\delta)/(3+\delta)}\bigr)+\frac{1}{\sqrt{2\pi}}\Biggl(\sum_{k=1}^{2}\gamma_{kn}^{1/3}+\gamma_{3n}^{\frac{2+\delta}{3+\delta}}\Biggr)\\\quad=\sum_{k=1}^{6} 克_{kn}。\结束{对齐}$$
(4.21)

因此,这足以表明\(\sum_{k=2}^{6} G公司_{kn}=O(\mu_{2n})\(G_{1n}=O(伽玛^{1/2}_{1n}+\gamma^{1/2}_{2n}+\upsilon_{2n}).

步骤1。我们首先证明\(\sum_{k=2}^{6} 克_{kn}=O(\mu_{2n}).类似于\(一)_{2n}-I_{8n}\)在定理中3.3,我们有

$$\开始{聚集}G_{2n}\leq\frac{E\vert J_{12n}\vert^{2}}{\gamma_{1n}^{2/3}}\leq \frac}{C}{\varGamma_{n}^}2}(t)\gamma_{1n{2/3}}\sum_{m=1}^{k}\sum_{i=l_{m}}^{l_{m}+q-1}\biggl(int_{A_{i}}E_{m}(t,s)\,ds\biggr)^{2}\sigma^{2}_{i} \leq\frac{Ckq2^{m}}{n\gamma{1n}^{2/3}}\leq C\gamma_{1n{1/3},\\G{3n}\leq\ frac{E\vert J{13n}\vert^{2}}{gamma{2n}^2,3}}\leq\frac{C}{varGamma_{n}^2}(t)\gamma_2n}^{2/3}}\sum_{i=k(p+q)+1}^{n}\biggl(int_{A{i}}E_{m}(t,s)\,ds\biggr)^{2}\sigma^{2}_{i} \leq\frac{C2^{m}p}{n\gamma_{2n}^{2/3}}\leq-C\gamma_2n}^}{1/3}。\结束{聚集}$$

注意,如果\(\xi_{n}\rightarrow\xi\sim n(0,1)\),然后\(E|\xi_{n}|\rightarrow E|\xi|=\sqrt{2/\pi}\)\(E|\xi_{n}|^{2+\delta}\rightarrow E|\xi|^{2+\ delta}\).根据定理3.3(i) 和(2.6)由此可见

$$\begin{aligned}&\vert\beta-E\hat{beta}_{L}\vert\leq E\vert\beta-\hat{beta{{L}\ vert=O\bigl(\sigma{1n}/S_{n}^{2}\bigr)=O\bigl(n^{-1/2}\biger),\end{alinged}$$
(4.22)
$$\begin{aligned}&E\vert\beta-\hat{\beta}_{L}\vert^{2+\delta}\leq O\bigl(\bigle(\sigma_{1n}/S_{n}^2}\bigr)^{2+/delta}\ bigr。\结束{对齐}$$
(4.23)

因此,应用Abel不等式(4.9)并将(A1)(iii)和(A2)(i)与引理结合答6,来自(4.22)和(4.23)我们得到

$$\beart{aligned}[b]\vert G_{4n}\vert&=\frac{1}{\sqrt{2\pi}\varGamma_{n}(t)}\cdot\vert\beta-E\hat{\beta}_{L}\vert\cdot\Biggl\vert\sum_{i=1}^{n} x个_{i} \int_{A_{i}}E_{m}(t,s)\,ds\Biggr\vert\\&\leq C\varGamma_{n}^{-1}(t)n^{-1/2}\Biggl vert\,ds\cdot\max_{1\leql\leqn}\Biggl\vert\sum_{i=1}^{l} v(v)_{j_{i}}\Biggr\vert\Biggr)\\&\leq C\bigl(2^{-m/2}+\sqrt{2^{m}/n}\log n\bigr)=C\gamma_{3n}\end{aligned}$$
(4.24)

$$开始{对齐}[b]\vert J_{3n}\vert^{2+\delta}&=\varGamma^{-(2+\delta)}_{n}(t)E\vert\beta-\hat{beta}_{L}\vert ^{2+/delta}\Biggl\vert\sum_{i=1}^{n} x个_{i} \int_{A_{i}}E_{m}(t,s)\,ds\Biggr\vert^{2+delta}\\&\leq C\varGamma_{n}^{-(2+delta)}l\vert E_{m}(t,s)\bigr\vert\,ds\cdot\max_{1\leq l\leq n}\Biggl\vert\sum_{i=1}^{l} v(v)_{j_{i}}\Biggr\vert\Biggr)^{2+\delta}\hsspace{-24pt}\\&\leq C\gamma^{2+/delta}_{3n},\end{aligned}$$
(4.25)

这意味着\(G{5n}\leqc\gamma{3n}^{(2+δ)/(3+δ)}).所以我们得到\(\sum_{k=2}^{6} G公司_{kn}=O(\mu_{2n}).

第2步。我们核实\(G{1n}=O(\gamma{1n{^{1/2}+\gamma_{2n}^{1/2}+\upsilon_{2n}).让\({\zeta{nm}:m=1,2,\ldots,k\})是独立的随机变量\(泽塔{nm})具有与相同的分布\(\chi_{nm}\),\(m=1,2,\ldot,k).设置\(T_{n}=\sum_{m=1}^{k}\ζ_{nm}\)\(t_{n}^{2}=\sum_{m=1}^{k}\操作符名{Var}(\chi_{nm})\).类似于(4.17),我们很容易看到

$$开始{对齐}[b]G_{1n}&=\sup_{y}\bigl\vert P(J_{11n}\leqy)-\varPhi(y)\bigr\vert\\&\leq\sup_y}\bigl\vertP(J_{11n}\leq y)-P/T_{n})\bigr\vert+\sup_{y}\bigl\vert\varPhi。\结束{对齐}$$
(4.26)

类似于(4.13)–(4.20),我们可以获得\(|t{n}^{2}-1|\列q C(\gamma_{1n}^{1/2}+\gamma_2n}^}1/2}+u(q)),\(|G_{12n}|\leq C\gamma_{2n}^{\delta/2}\),\(|G_{13n}|\leq C(\gamma_{1n}^{1/2}+\gamma_2n}^_1/2}+u(q))、和\(|G_{11n}|\leq C\upsilon _{2n}\)。由此可见\(G{1n}=O(\gamma{1n{^{1/2}+\gamma_{2n}^{1/2}+\upsilon_{2n}).定理证明3.4已完成。 □

推论的证明3.2

出租\(p=\lfloor n^{\rho}\rfloor),\(q=地板n^{2\rho-1}\rfloor),\(增量=1),何时\(1/2<ρ<θ<1),我们有\(\gamma^{1/3}_{1n}=O(n^{-(θ-\rho)/3}),\(\gamma^{1/3}_{2n}=O(n^{-(θ-\rho)/3}),\(伽玛^{3/4}_{3n}=O(n^{-3(1-\θ)/8})、和\(u^{1/3}(q)=O(n^{-(θ-\rho)/3}).因此(3.8)直接遵循定理3.4. □

工具书类

  1. Engle,R.,Granger,C.,Rice,J.,Weiss,A.:天气和电力销售之间关系的半参数估计。美国统计协会。81,310–320(1986年)

    第条 谷歌学者 

  2. Gao,J.T.,Chen,X.R.,Zhao,L.C.:部分线性模型中一类估计量的渐近正态性。数学学报。罪。37(2), 256–268 (1994)

    数学科学网 数学 谷歌学者 

  3. Chen,M.H.,Ren,Z.,Hu,S.H.:部分线性模型中一类估计的强相合性。数学学报。罪。41(2), 429–439 (1998)

    数学科学网 数学 谷歌学者 

  4. Hamilton,S.A.,Truong,Y.K.:部分线性模型中的局部线性估计。J.多变量。分析。60, 1–19 (1997)

    第条 数学科学网 谷歌学者 

  5. Härdle,W.,Liang,H.Y.,Gao,J.L.:偏线性模型。Physica-Verlag,海德堡(2000)

     谷歌学者 

  6. Liang,H.Y.,Mammitzsch,V.,Steinebach,J.:半参数回归模型,其误差来自具有负相关创新的线性过程。统计40(3), 207–226 (2006)

    第条 数学科学网 谷歌学者 

  7. Liang,H.Y.,Fan,G.L.:具有线性过程误差的半参数模型中估计量的Berry–Esséen型界。J.多变量。分析。100, 1–15 (2009)

    第条 谷歌学者 

  8. Liang,H.Y.,Jing,B.Y.:基于相依误差的部分线性模型中的渐近正态性。J.统计计划。推断139, 1357–1374 (2009)

    第条 数学科学网 谷歌学者 

  9. You,J.,Chen,G.:具有相关误差的部分线性回归模型中的半参数广义最小二乘估计。数学学报。罪。英语。序列号。23(6), 1013–1024 (2007)

    第条 数学科学网 谷歌学者 

  10. Back,J.I.,Liang,H.Y.:NA样本下半参数模型中估计量的渐近性。J.统计计划。推断136,3362–3382(2006年)

    第条 数学科学网 谷歌学者 

  11. Zhang,J.J.,Liang,H.Y.:异方差半参数模型中的Berry–Esséen型界。J.统计计划。推断141, 3447–3462 (2011)

    第条 谷歌学者 

  12. Wei,C.D.,Li,Y.M.:具有线性过程误差的半参数回归模型中小波估计的Berry–Esséen界。J.不平等。申请。2012,文章ID 44(2012)

    第条 谷歌学者 

  13. Robinson,P.M.:未知形式异方差存在下的渐近有效估计。计量经济学55, 875–891 (1987)

    第条 数学科学网 谷歌学者 

  14. Caroll,R.J.,Härdle,W.:半参数加权最小二乘回归中的二阶效应。统计2, 179–186 (1989)

    数学科学网 数学 谷歌学者 

  15. Liang,H.Y.,Qi,Y.Y.:NA假设下回归函数小波估计的渐近正态性。牛市。韩国数学。Soc公司。44(2), 247–257 (2007)

    第条 数学科学网 谷歌学者 

  16. Antoniadis,A.、Gregoire,G.、Mckeague,I.W.:曲线估计的小波方法。美国统计协会。89, 1340–1352 (1994)

    第条 数学科学网 谷歌学者 

  17. Sun,Y.,Chai,G.X.:固定设计回归函数的非参数小波估计。数学学报。科学。24安(5), 597–606 (2004)

    数学科学网 数学 谷歌学者 

  18. Li,Y.M.,Yang,S.C.,Zhou,Y.:具有相关样本的回归模型中小波估计的一致性和一致渐近正态性。统计概率。莱特。78, 2947–2956 (2008)

    第条 数学科学网 谷歌学者 

  19. Li,Y.M.,Guo,J.H.:强混合误差下小波估计的渐近正态性。J.韩国统计学会。38, 383–390 (2009)

    第条 数学科学网 谷歌学者 

  20. Li,Y.M.,Wei,C.D.,Xing,G.D.:线性过程误差回归模型中小波估计量的Berry–Esséen界。统计概率。莱特。81(1), 103–110 (2011)

    第条 谷歌学者 

  21. Xue,L.G.:混合误差下回归函数小波估计的一致收敛速度。数学学报。科学。22个(4) ,528–535(2002年)

    数学科学网 数学 谷歌学者 

  22. Zhou,X.C.,Lin,J.G.,Yin,C.M.:弱相关过程非参数回归模型中基于小波估计的渐近性质。J.不平等。申请。2013,文章ID 261(2013)

    第条 谷歌学者 

  23. Alam,K.,Saxena,K.M.L.:多元分布的正相关性。Commun公司。统计、理论方法10, 1183–1196 (1981)

    第条 数学科学网 谷歌学者 

  24. Joag-Dev,K.,Proschan,F.:随机变量与应用的负关联。Ann.统计。11(1), 286–295 (1983)

    第条 数学科学网 谷歌学者 

  25. Matula,P.:关于负相关随机变量和的几乎必然收敛性的注记。统计概率。莱特。15, 209–213 (1992)

    第条 谷歌学者 

  26. Shao,Q.M.,Su,Q.:负相关随机变量的重对数定律。斯托克。过程。应用。83, 139–148 (1999)

    第条 数学科学网 谷歌学者 

  27. Liang,H.Y.,Li,Y.Y.:基于线性过程误差的回归估计量的Berry–Esséen型界。J.韩国数学。Soc公司。45(6), 1753–1767 (2008)

    第条 数学科学网 谷歌学者 

  28. Petrov,V.V.:概率论的极限定理:独立随机变量序列。牛津大学出版社,纽约(1995)

    数学 谷歌学者 

  29. Yang,S.C.:负相关样本回归加权估计的一致渐近正态性。统计概率。莱特。62(2), 101–110 (2003)

    第条 数学科学网 谷歌学者 

下载参考资料

致谢

作者感谢编辑和审稿人的宝贵意见和建议,这些意见和建议提高了我们的论文质量。

基金

本研究部分得到了国家自然科学基金项目(No.11626031)和安徽省自然科学基金(KJ2016A428)的支持。

作者信息

作者和附属机构

作者

贡献

所有作者对本文的写作贡献均等。所有作者阅读并批准了最终手稿。

通讯作者

与的通信胡雪萍.

道德声明

竞争性利益

作者声明,他们没有相互竞争的利益。

其他信息

出版商备注

Springer Nature在公布的地图和机构关联中的管辖权主张方面保持中立。

附录

附录

引理A.1

(背和梁[10])

\({X_{n},n\geq1\})是具有零均值的NA随机变量序列 \(阿尔法>2).假设 \({a{ni},1\leqi\leqn,n\geq1\})是实数的三角形数组 \(\最大_{1\leq i \ leq n}| a_{ni}|=O(n ^{-1/2})\) \(\sum_{i=1}^{n} 一个_{ni}^{2}=o(n^{-2/\alpha}(\logn)^{-1}).如果 \(\sup_{i}\mathrm{E}|X_{i{|^{p}<\infty\)对一些人来说 \(p>2α/(α-2)),然后 \(\sum_{i=1}^{n} 一个_{ni}X_{i} =o(n^{-1/\alpha})\)..

引理A.2

(背和梁[10])

\({X_{n},n\geq1\})是平均值为零的NA随机变量序列.假设 \({a{ni},1\leqi\leqn,n\geq1\})是在闭合区间上定义的函数数组属于R(右)令人满意的 \(最大{1\leqi,j\leqn}|a{ni}(u{j})|=O(n^{-1/2})\) \(最大{1\leqj\leqn}\sum{i=1}^{n}|a{ni}(u{j})|=O(1)\).如果 \(\sup_{i}\mathrm{E}|X_{i{|^{p}<\infty\)对一些人来说 \(p>2),然后

$$\begin{collected}{(\mathrm{i})}\quad\max_{1\leqj\leqn}\Biggl\vert\sum_{i=1}^{n} 一个_{ni}(u_{j})X_{i}\Biggr\vert=o\bigl,\结束{聚集}$$

哪里 \(L(x)>0)是一个缓慢变化的函数 \(x\rightarrow\infty\), \(\sqrt{x} L(左)(x) \)不会减少 \(x\geqx{0}>0\).

引理A.3

(梁和李[27])

\({X_{n};n\geq1\})是具有零均值的NA随机变量序列 \(\mathrm{E}|X_{n}|^{p}<\infty\)对一些人来说 \(p>1),然后让 \({b{i},i\geq1\})是实数序列.然后存在一个正常数 \(C_{p}\)这样的话

$$\mathrm{E}\max_{1\leq m\leq n}\Biggl\vert\sum_{i=1}^{m} b条_{i} X(X)_{i} \Biggr\vert^{p}\leq C_{p}\Biggl\{\sum_{i=1}^{n}\mathrm{E}\vert b_{i} X(X)_{i} \vert^{p}+i(p>2)\Biggl(\sum_{i=1}^{n}\mathrm{E}(b_{i} X(X)_{i} )^{2}\Biggr)^{p/2}\Biggr\}$$

引理A.4

(杨[29])

假设 \({\varsigma_{n},n\geq1\}),\({\eta_{n},n\geq1\}), \({\xi_{n},n\geq1\})是三个随机变量序列,\({\gamma_{n},n\geq1\})是一个正的非随机序列, \(\gamma_{n}\rightarrow0\).如果 \(\sup_{x}|F_{\varsigma_{n}}(x)-\varPhi(x)|\leq C\gamma_{n{),那么对于任何 \(\varepsilon_{1}>0\) \(\varepsilon_{2}>0\),

$$\sup_{x}\bigl\vert F_{\varsigma_{n}+\eta_{n}+\xi_{n{}}(x)-\varPhi(x)\bigr\vert\leq C\bigl \{gamma_{n{+\varepsilon_{1}+\varesilon_{2}+P\bigl.\geq\varepsilon_{2}\bigr)\bigr\}$$

引理A.5

(梁和凡[7])

假设 \(X_{n}\),\(第1页)是具有有限二阶矩的NA随机变量序列. \({a{j},j\geq1\})成为一个真实的序列,然后让 \(1=m{0}<m{1}<cdots<m{k}=n\).定义 \(Y{l}=\sum{j=m{l-1}+1}^{m{l}}_{j} X(X)_{j} \)对于 \(1\leq l\leq k\).然后

$$\Biggl\vert E\exp\Biggl \{\mathrm{i} t吨\sum_{l=1}^{k} Y(Y)_{l} \Biggr\}-\prod_{i=1}^{k}\exp\{\mathrm{i} tY型_{l} \}\Biggr\vert\leq4t^{2}\sum_{1\leqs<j\leqk}\sum_{l_{1}=m_{s-1}+1}^{m_{s}}\sum_a{2}=m_a{j-1}+1}},X_{l_{2}})\bigr\vert$$

引理A.6

(魏和李[12])

假设(A3)(A4)持有.然后

  1. (i)

    \(\sup_{m}\int_{0}^{1}|\mathrm{电子}_{m} (t,s)|\,ds\leq C\);

  2. (ii)

    \(\sum_{i=1}^{n}|\int_{A{i}}\mathrm{电子}_{m} (t,s)\,ds | \leq C\);

  3. (iii)

    \(\sup_{0\leqs,t\leq1}|\mathrm{电子}_{m} (t,s)|=O(2^{m})\);

  4. (iv)

    \(|\int_{A_{i}}\mathrm{电子}_{m} (t,s)\,ds |=O(\压裂{2^{m}}{n})\),\(i=1,2,\ldot,n);

  5. (v)

    \(sum_{i=1}^{n}(int_{A{i}}){电子}_{m} (t,s)\,ds)^{2}=O(\分形{2^{m}}{n})\);

  6. (vi)

    \(max_{1\leqi\leqn}\sum_{j=1}^{n}\int_{A{j}}|\mathrm{电子}_{m} (t_{i},s)|\,ds\leq C\);

  7. (vii)

    \(max_{1\leqi\leqn}\sum_{j=1}^{n}\int_{A{i}}|\mathrm{电子}_{m} (t_{j},s)|\,ds\leq C\).

权利和权限

开放式访问本文根据知识共享署名4.0国际许可证获得许可,该许可证允许以任何媒介或格式使用、共享、改编、分发和复制,只要您对原作者和来源给予适当的信任,提供知识共享许可证的链接,并说明是否进行了更改。本文中的图像或其他第三方材料包含在文章的Creative Commons许可证中,除非材料的信用额度中另有说明。如果文章的知识共享许可证中没有包含材料,并且您的预期用途不被法律法规允许或超出了允许的用途,则您需要直接获得版权所有者的许可。要查看此许可证的副本,请访问http://creativecommons.org/licenses/by/4.0/.

转载和许可

关于本文

检查更新。通过CrossMark验证货币和真实性

引用这篇文章

胡,X.,钟,J.,任,J。等。基于负相关新息的异方差半参数模型中小波估计的渐近性质。J不平等申请 2019, 314 (2019). https://doi.org/10.1186/s13660-019-2267-4

下载引文

  • 收到以下为:

  • 认可的以下为:

  • 已发布以下为:

  • 内政部以下为:https://doi.org/10.1186/s13660-019-2267-4

移动交换中心

关键词